The points scored on a test for a sample of 39 students are summarized in the following table

The Points Scored On A Test For A Sample Of 39 Students Are Summarized In The Following Table

Answers

Answer 1

the mean score of the test for sample of 39 students is 76.92.

The data is given as:

Number of students       Score of each student       Total score

            07                                       90                              630

             17                                       80                             1360

             11                                        70                               770

            04                                       60                              240

The total number of students = 7 + 17 + 11 + 4

n = 39

Total scores = 630 + 1360 + 770 + 240

T = 3000

Mean score = T / n

M = 3000 / 39

M = 1000 / 13

M = 76.92

Therefore, the mean score of the test for sample of 39 students is 76.92.

Learn more about mean here:

https://brainly.com/question/1136789

#SPJ9


Related Questions

Drag the tiles to the correct boxes to complete the pairs. Not all tiles will be used. Match each series with the equivalent series written in sima notation.

Answers

Step 1

Given;

Step 2

[tex]3(5)^0+3(5)^1+3(5)^2+3(5)^3+3(5)^4[/tex][tex]3+15+75+375+1875[/tex][tex]\begin{gathered} 4(8)^0+4(8)^1+4(8)^2+4(8)^3+4(8)^4 \\ 4+32+256+2048+16348 \end{gathered}[/tex][tex]\begin{gathered} 2(3)^0+2(3)^1+2(3)^2+2(3)^3+2(3)^4 \\ 2+6+18+54+162 \end{gathered}[/tex][tex]\begin{gathered} 3(4)^0+3(4)^1+3(4)^2+3(4)^3+3(4)^4 \\ 3+12+48+192+768 \end{gathered}[/tex]

Answer:

Jennifer uses a coupon that gives you20% off your order. If the total was$18, how much money did she save?

Answers

let M be the money, hence, she saved

[tex]\begin{gathered} (0.2)\cdot18=3.6\text{ dollars} \\ \\ \end{gathered}[/tex]

of the 800 participants in a marathon, 120 are running to raise money for a cause. How many participants out of 100 are running for a cause?a.8 b. 20c. 15d. 12OMG i hate iready please heeeelp

Answers

To find how many participants out of 100 are running for a cause we can use the next proportion:

[tex]\frac{800\text{ total participants}}{100\text{ total participants}}=\frac{120\text{ running for a cause}}{x\text{ running for a cause}}[/tex]

Solving for x:

[tex]undefined[/tex]

The salesperson earned a commission of $1110.20 for selling $7930 worth of paper products. Find the commission rate

Answers

Commision = $1110.20

Selling= $7930

x is the commission rate

[tex]7930\cdot\frac{x}{100}=1110.20[/tex]

Then we isolate the x

[tex]x=\frac{1110.20\cdot100}{7930}=14\text{ \%}[/tex]

ANSWER

The commission rate is 14%

HELPPPP MEEEEEE PLEASEEEEhey tutor how you doing doing I struggle with math so much.

Answers

Answer:

[tex]m\measuredangle8=110^o[/tex]

Explanation:

The angles 4 and 8 are equal; therefore,

[tex]m\measuredangle8=m\measuredangle4[/tex][tex]3x+20=x+80[/tex]

Subtracting x from both sides gives

[tex]2x+20=80[/tex]

Subtracting 20 from both sides gives

[tex]2x=80-20[/tex][tex]2x=60[/tex]

Finally, dividing both sides by 2 gives

[tex]\boxed{x=30.}[/tex]

With the value of x in hand, we now find the measure of angle 8.

[tex]m\measuredangle8=x+80[/tex][tex]m\measuredangle8=30+80[/tex][tex]\boxed{m\measuredangle8=110^o\text{.}}[/tex]

Hence, the measure of angle 8 is 110.

Dustin boat traveled 36 miles downstream in three hours. The same boat traveled 30 miles upstream in five hours. What is the speed of the boat and the speed of the current

Answers

Answer:

The speed of the boat is 9 miles/hour and the speed of the current is 3 miles/hour

Explanation:

Let's call x the speed of the boat and y the speed of the current.

The distance traveled is equal to the speed times the time, so the boat traveled 36 miles in three hours and we can write the following equation

(x + y)3 = 36

3(x + y) = 36

because when the boat traveled downstream, the total speed is the sum of x and y.

On the other hand, the boat traveled 30 miles upstream in 5 hours, so

(x - y)5 = 30

5(x - y) = 30

Therefore, the system of equations is

3(x + y) = 36

5(x - y) = 30

Solving the first equation for x, we get

[tex]\begin{gathered} 3(x+y)=36 \\ \\ \frac{3(x+y)}{3}=\frac{36}{3} \\ \\ x+y=12 \\ x+y-y=12-y \\ x=12-y \end{gathered}[/tex]

Now, we can replace this expression on the second equation as follows

[tex]\begin{gathered} 5(x-y)=30 \\ \\ {\frac{5(x-y)}{5}}=\frac{30}{5} \\ \\ x-y=6 \\ \\ \text{ Replacing x = 12 - y} \\ 12-y-y=6 \\ 12-2y=6 \\ 12-2y-12=6-12 \\ -2y=-6 \\ \\ \frac{-2y}{-2}=\frac{-6}{-2} \\ \\ y=3 \end{gathered}[/tex]

Then, the value of x is

x = 12 - y

x = 12 - 3

x = 9

So, the speed of the boat is 9 miles/hour and the speed of the current is 3 miles/hour

J is the midpoint of HK . What are HJ, JK, and HK?

Answers

HJ=25

JK=25

HK=50

Explanation

Step 1

J is the midpoint, it means

[tex]HJ=JK[/tex]

Step 2

replace andsolve for x

[tex]\begin{gathered} HJ=JK \\ 9x-2=4x+13 \\ \text{subtract 4x in both sides} \\ 9x-2-4x=4x+13-4x \\ 5x-2=13 \\ add\text{ 2 in both sides} \\ 5x-2+2=13+2 \\ 5x=15 \\ divide\text{ both sides by 5} \\ \frac{5x}{5}=\frac{15}{5} \\ x=3 \\ \end{gathered}[/tex]

Step 3

finally replace the valure of X to find HJ and JK

[tex]\begin{gathered} HJ=JK=9x-2=9\cdot3-2=27-2=25 \\ HJ=25 \\ JK=25 \\ then \\ HK=HJ+JK=25+25 \\ \\ HK=50 \end{gathered}[/tex]

I hope this helps you

Select the correct answer. What is the difference of the values of the two variables in this system of equations? y= 2x + 1 x + 3y = 10 O A. 0 B. 1 KD C. 2 KD D. 3

Answers

According to the given data we have the following equation:

2x + 1 x + 3y = 10

There are two types of variables in the equation above.

The variable x and the variable y

In order to calculate the difference of the values of the two variables we would make the following:

First we would sum elements of variable x

variable x=2x + 1x=3x

variable y=3y

Therefore, the difference of the values=3x-3y=0

So, The right answer would be A, the value is 0.


Marc se come un sándwich de huevo para el desayuno y una hamburguesa grande para el almuerzo todos los días.
El sándwich de huevo tiene 250 calorías. Si Marc come 5,250 calorías en el desayuno y almuerzo en toda la
semana en total, ¿cuántas calorías tiene una hamburguesa grande?
lón de juegos la primera vez ella ganó 60 boletos. La segunda vez,

Answers

Answer:Hay 500 calorías en una Big Burger.

Step-by-step explanation:

En una semana (7 días), Mark come 7 sándwiches de huevo, que son 1750 calorías. Reste la cantidad total de calorías que consumió por la cantidad de calorías consumidas a través de sándwiches de huevo; 5250-1750=3500. 3500 es el número total de calorías que Mark consumió al comer una Big Burger todos los días durante 7 días. Divide 3500 entre 7 = 500. Hay 500 calorías en una Big Burger.

1. An account is opened with a balance of $2800earning 4.25% simple interest. What will be thebalance in the account in 30 years?

Answers

Answer:

$6370

Explanation:

The simple interest formula gives us the final amount A given the principal amount P:

[tex]A=P(1+rt)[/tex]

where r is the interest rate and t is the time interval.

Now in our case we have

P = 2800

r = 4.25/100

t = 30 years

therefore, the above formula gives

[tex]A=2800(1+\frac{4.25}{100}\cdot30)[/tex]

which simplifies to give

[tex]\boxed{A=\$6370}[/tex]

Hence, the account balance after 30 years will be $6370.

Find the 38th term 359,352,345

Answers

Let's begin by listing out the information given to us:

1st term = 359, 2nd term = 352, 3rd term = 345

[tex]\begin{gathered} 359,352,345\ldots x_n \\ x_1=359,x_2=352,x_3=345 \\ x_1-x_2=x_2-x_3\Rightarrow359-352=352-345\Rightarrow7=7 \\ 7=7 \end{gathered}[/tex]

This is an Arithmetic Progression (A.P.)

[tex]\begin{gathered} x_1=359 \\ x_2=359-7(2-1)\Rightarrow359-7(1)=359-7=352 \\ x_3=359-7(3-1)\Rightarrow359-7(2)=359-14=345 \\ x_n=x_1-7(n-1) \\ n_{38}=x_1-7(38-1)=359-7(37)=359-259=100 \\ n_{38}=100 \end{gathered}[/tex]

Julie is buying chocolate chip and oatmeal cookies from the bakery. Chocolate chip cookies cost 25¢ each and oatmeal cookies cost 20c each. She wants to buy a mixture of at least 50 cookies. Julie is planning to spend less than $10. Let: C = number of chocolate chip cookies she can buy. M = number of oatmeal cookies she can buy. Select the system of inequalities that represents this situation.

Answers

[tex]\begin{gathered} \text{ Since we have to spend less that \$10} \\ 0.25C+0.20M<10.00 \\ \\ \text{ Since she want to buy at least 50,} \\ C+M\ge50 \\ \\ \text{ The answer is } \\ 0.25C+0.20M<10.00 \\ C+M\ge50| \end{gathered}[/tex]

What is the reason these triangles are congruent? M N Р o Not Congruent

Answers

Since the line in red is common to both triangles and segments PM and ON are parallel, then the angles in purple are congruent and so are the angles in green. So they are congruent by ASA

Find the volume of a rectangular prism with the following dimensions.length: 4.2 cmwidth: 7 cmheight: 15 cmvolume = ____ cm3

Answers

Given:

length: 4.2 cm

width: 7 cm

height: 15 cm

Required:

volume = ____ cm3

Explanation:

volume of prism=

[tex]\begin{gathered} l\times w\times h \\ 4.2\times7\times15 \\ =441cm^3 \end{gathered}[/tex]

Required answer:

[tex]441cm^3[/tex]

Sue receives $7 per hour when she works at the book store. Last week she earned $259.How many hours did she work at her job?

Answers

1 hour = $7

Number of hours = Amount/7

Therefore, 259/7 = 37 hours

Solution: Sue worked for 37 hours last week.

how do i find out if a table is a linear function? i know the formula i just dont know how to figure out if its linear, thanks!

Answers

Answer:

Table 3

Explanation:

A linear function has a constant slope.

To determine if the table represents a linear function, find the slope for two different pairs of points.

Table 1

Using the points (1,-2), (2,-6)

[tex]\text{Slope},m=\frac{Change\text{ in y-axis}}{Change\text{ in x-axis}}=\frac{-6-(-2)}{2-1}=-6+2=-4[/tex]

Using the points (2,-6), (3,-2)

[tex]\text{Slope},m=\frac{Change\text{ in y-axis}}{Change\text{ in x-axis}}=\frac{-2-(-6)}{3-2}=-2+6=4[/tex]

The slopes are not the same, thus, the function is not linear.

Table 3

Using the points (1,-2), (2,-10)

[tex]\text{Slope},m=\frac{Change\text{ in y-axis}}{Change\text{ in x-axis}}=\frac{-10-(-2)}{2-1}=-10+2=-8[/tex]

Using the points (2,-10), (3,-18)

[tex]\text{Slope},m=\frac{Change\text{ in y-axis}}{Change\text{ in x-axis}}=\frac{-18-(-10)}{3-2}=-18+10=-8[/tex]

The slopes are the same, thus, the function is linear.

Table 3 is the correct option.

The Dover Symphony categorizes its donors as gold, silver, or bronze depending on the amount donated.

Answers

Explanation

Given the donors as

[tex]\begin{gathered} Gold=4 \\ silver=7 \\ Bronze=9 \end{gathered}[/tex]

The total number of donors are

[tex]9+7+4=20[/tex]

Therefore, the percent of donors at the bronze or silver level is

[tex]\frac{sum\text{ }of\text{ }bronze\text{ }and\text{ }silver\text{ donors}}{number\text{ of donors}}=\frac{9+7}{20}\times100=16\times5=80\text{\%}[/tex]

Answer:

The graph of y= 2x^2 - kx + 6 touches the x-axis. What are the possible value(s) of k?

Answers

Given:

The graph of

[tex]y=2x^2-kx+6[/tex]

Required:

What are the possible value(s) of k?

Explanation:

[tex]Set\text{ y = 0, evaluate the quadratic at }h=-\frac{b}{2a}and\text{ solve for k}[/tex]

You want to find the value the value of k such that the y coordinate of the vertex is 0.

[tex]\begin{gathered} y=2x^2-kx+6 \\ 0=2x^2-kx+6 \end{gathered}[/tex]

The x coordinate, h , of the vertex is found, using the following equation:

[tex]\begin{gathered} D=b^2-4ac \\ b^2-4ac=0 \\ k^2-4\times2\times6=0 \\ k^2-48=0 \\ k^2=48 \\ k=\pm4\sqrt{3} \end{gathered}[/tex]

Answer:

So, values of k are above.

T-Mobile charges a flat fee of $20 plus $10 per Gig of data used per month. AT&T charges $60 for an unlimiteddata use. How many Gigs of data would you have to use so that the cost will be the same for both companies?

Answers

For this case we can set uo an equation given by:

[tex]y=10x+20[/tex]

Where y represent the final cost. x the number of Gig used and for this case we can set up the following equation:

[tex]60=10x+20[/tex]

And solving for x we got:

[tex]x=\frac{60-20}{10}=\frac{40}{10}=4[/tex]

And the final answer for this case woudl be 4 Gig of data used

Need help solving question 34 via expanding and simplifying thanks

Answers

34. The equation is given as

[tex](x+y)^2-x(2-y)[/tex]

Solving the equation by expanding and simplifying.

Use the identity,

[tex](a+b)^2=a^2+b^2+2ab[/tex][tex]x^2+y^2+2xy-2x+xy[/tex][tex]x^2+y^2-2x+3xy[/tex]

Hence the answer is

[tex]x^2+y^2-2x+3xy[/tex]

Assume that random guesses are made for six multiple-choice questions on a test with five choices for each question so that there are n equals six trials each with the probability of success (correct) given by P equals 0.20. Find the probability of no correct answers.

Answers

Given in the question:

a.) Random guesses are made for six multiple-choice questions.

b.) There are five choices for each question.

c.) There are n equals six trials each with the probability of success (correct) given by P equals 0.20.

We will be using the Binomial Probability Formula:

[tex]P(X=k)=(_nC_k)(p^k)(1-p)^{n-k}[/tex]

Where,

n = Number of trials = 6

P = Probability of success = 0.20

X = Correct answers

Let's evaluate the definition of binomial probability at k = 0 since we are tasked to find the probability of no correct answers.

[tex]P(X=0)=(_6C_0)(0.20^0)(1-0.20)^{6-0}[/tex][tex]P(X=0)\text{ = (}\frac{6!}{0!(6-0)!})(0.20^0)(0.80^6)^{}^{}[/tex][tex]P(X=0)\text{ = }0.262144\text{ }\approx\text{ 0.26}2[/tex]

Therefore, the probability of no correct answers is 0.262 or 26.20%.

In the relationship shown by the data linear ? If so , model the data with an equation A. The relationship is not linear B. The relationship is linear; y+2=4/5 (x+9) C . The relationship is linear; y + 9 = - 4/5 (x+2) D. The relationship is linear; y+ 2 = -5/4 (x+9)

Answers

Let's take two points so that we can get the equation of the line which goes through those points. P1 (-9, -2), P2 (3, -17):

[tex]m=\frac{y_2-y_1}{x_2-x_1}=\frac{-17-(-2)}{3-(-9)}=\frac{-17+2}{3+9}=-\frac{15}{12}=-\frac{5}{4}[/tex][tex]\begin{gathered} y-y_1=m(x-x_1) \\ y-(-2)=-\frac{5}{4}\cdot(x-(-9)) \\ y+2=-\frac{5}{4}\cdot(x+9)_{} \\ y=-\frac{5}{4}x-\frac{45}{9}-2 \\ y=-\frac{5}{4}x-7 \\ f(x)=-\frac{5}{4}x-7 \end{gathered}[/tex]

So, y is the line which goes through the first and last points of the chart.

To proof that the rest of points go through the line as well, we will evalute each point

[tex]\begin{gathered} f(-5)=-\frac{5}{4}\cdot(-5)-7=\frac{25}{4}-7=-\frac{3}{4}\ne-7 \\ f(-1)=-\frac{5}{4}(-1)-7=\frac{5}{4}-7=-\frac{23}{4}\ne-12 \end{gathered}[/tex]

Since the evaluation of these points don't correspond to the values of the chart we can assure that the relationship is not linear

Erika is working on solving the exponential equation 50^x = 17; however, she is not quite sure where to start. Using complete sentences, describe to Erika how to solve this equation.

Answers

In this case, we'll have to carry out several steps to find the solution.

Step 01:

Data:

50^x = 17

Step 02:

exponential equation:

1. Apply logarithms to both sides of the equality.

[tex]log\text{ 50}^x=\text{ log 17}[/tex]

2. Apply properties of logarithms.

[tex]x\text{ log 50 = log 17}[/tex]

3. Apply the algebraic rules to find the value of x.

[tex]x\text{ = }\frac{log\text{ 17}}{log\text{ 50}}\text{ }[/tex]

The answer is:

x = (log 17) / (log 50)

Solve each systems of the equations by elimination. 1* x-y=-13 x+y=-52* 2x-9y=17 2x+3y=-19

Answers

Let us solve the given system of equations by using the elimination method.

Question 1:

[tex]\begin{gathered} x-y=-13\quad eq.1 \\ x+y=-5\quad eq.2 \end{gathered}[/tex]

Add these two equations so that the y variable cancels out

So, the value of x can be found now

[tex]\begin{gathered} 2x=-18 \\ x=-\frac{18}{2} \\ x=-9 \end{gathered}[/tex]

Substitute the value x into any of the two equations to find the value of y.

[tex]\begin{gathered} x-y=-13 \\ -9-y=-13 \\ y=-9+13 \\ y=4 \end{gathered}[/tex]

Therefore, the solution of this system is x = -9 and y = 4

A skating rink attendant monitored the number of injuries at the rink over the past year. He tracked the ages of those injured and the kinds of skates worn during injury. In-line skates Roller skates Age 8 11 10 Age 10 4 9 Age 12 3 16 What is the probability that a randomly selected injured skater was not age 12 and was not wearing roller skates? Simplify any fractions.

Answers

Given data:

The given table is shown.

The expression for the probability of that a randomly selected injured skater was not age 12 and was not wearing roller skates is,

[tex]undefined[/tex]

-(1 – 7a) = 3(8a - 6)

Answers

[tex]\begin{gathered} -(1-7a)=3(8a-6) \\ \text{Multiply the left parenthesis by negative 1} \\ -1+7a=3(8a-6) \\ \text{Next you multiply the right parenthesis by 3} \\ -1+7a=24a-18 \\ \text{Collect all like terms, which means;} \\ \text{The unknown terms will move to one side of the equation and} \\ \text{the known numbers will move to the other side} \\ -1+18=24a-7a \\ \text{Note that when a positive value crosses to the other side of an equation} \\ it\text{ becomes negative, and vice versa for a negative value} \\ 17=17a \\ \text{Divide both sides by 17} \\ \frac{17}{17}=\frac{17a}{17} \\ 1=a \end{gathered}[/tex]

diana and her classmates are reading the same book.on Monday, diana started to write down the number of pages she has left to read at the end of each day from Monday through Thursday, which person is reading the same number of pages per day as diana.

Answers

From the diana table we can conclude:

[tex]\begin{gathered} x1=187 \\ x2=181 \\ x3=175 \\ x4=169 \\ x2-x1=x4-x3=6 \end{gathered}[/tex]

She's reading 6 pages per day.

Since:

[tex]\begin{gathered} y1=180 \\ y2=174 \\ y3=168 \\ y4=162 \\ y2-y1=y4-y3=6 \end{gathered}[/tex]

Keith is also reading 6 pages per day.

1 1 2. Consider 2 divided by 2 (a) Write a real-world problem for the division. (b) Create a model or write an equation for the division. (C) Find the quotient for the real-world problem in part (a). Show your work or explain your reasoning. Answer:

Answers

We will have the following:

a) His parents spent:

[tex]2.49\cdot6=14.94[/tex]

So they spent $19.94.

b) They will spent the following in rental:

[tex]\frac{150}{8}=18.75[/tex]

So, the hourly rate $18.75.

c) We will determine the amount spent:

[tex]182.53-150-14.49=18.04[/tex]

So, it would be $18.04.

The average temperature on the planet A is 162°C. Convert this temperature to degrees Fahrenheit. Round to the nearestdegreeUse the formula F =+ 32162° Celsius is equivalent toFahrenheit.

Answers

Using the formula:

[tex]F=\frac{9}{5}c+32[/tex]

We get:

[tex]\begin{gathered} F=\frac{9}{5}(162)+32 \\ F=323.6 \end{gathered}[/tex]

162°C is equal to 323.6 F

A homeowner has decided to fill in his pool. The pool is rectangular and measures 20ft wide, 40ft long, and 5.5ft deep throughout. Each cubic yard of fill dirt cost $12. How much will it cost to fill the pool?

Answers

The volume of the pool is

[tex]20ft\text{ }\times40ft\text{ }\times5.5ft=4400ft^3[/tex]

a cubic foot is 0.037 cubic yards.

thus

[tex]4400ft^{3^{}}^{}=4400\times0.037=162.8yd^3[/tex]

but a cubic yard of dirt costs $12, and we need 162.8 cubic yards.

that would cost

[tex]12\times162.8=\text{ \$1953.6}[/tex]

Other Questions
Please help tonight!!! 50 points! Two objects producing 0.004 N of gravitational force one object had a mass of 3000 kg in another object had a mass of 2600 kg what is the distance between them Nora has a coupon for $3 off of a calzone. She orders a beef and olive calzone, and her bill, with the discounted price is $9.49. What is the regular price of the calzone? Make sure to round your answer to the nearest cent. Do not place a dollar sign as it will not be needed for this question. during december, vixen company sells $863,000 in merchandise that has a one-year warranty. warranty expense is estimated at 3% of sales. on january 5 of the following year, the merchandise requires repairs that are completed the same day. the repairs cost $15,300 for materials taken from parts inventory. the entry to record the repairs that occur on january 5 is: The taxes on a house assessed at $71,000 are $1775 a year. If the assessment is raised to $114,000 and the tax rate did not change, how much would thetaxes be now? A 0.4 kg object is attached to a spring witha spring constant 145 N/m so that the objectis allowed to move on a horizontal frictionlesssurface. The object is released from rest whenthe spring is compressed 0.14 m.Find the force on the object when it isreleased.Answer in units of N. Two consecutive terms in an ARITHMETIC sequence are given. Find the recursive function. If Ashley had 4 yards of yarn and Ramon had 11 feet of yarn, who had more yarn? Hey I need help on this math problem ignore the lines across the answer choices its a glitch I cant change it and the lines dont mean that the answer choice is wrong In Millersburg, the use of landlines has been declining at a rate of 10% every year. If there are 42,000 landlines this year, how many will there be in 7 years?If necessary, round your answer to the nearest whole number. this is a 4 question part which price has the lowest unit per ounce choice a 6 ounces of chocolate chips for $ 2.49 choice b 8 ounces of chocolate chips for $ 3.32 I will ask the other 3 questions soon Refer to the table which summarizes the results of testing for a certain disease. A test subject is randomly selected and tested for the disease. What is the probability the subject has the disease given that the test result is negative. Round to three decimal places as needed.Positive Test ResultNegative Test ResultSubject has the disease879Subject does not have the disease27312 The American Colonist became more and more unhappy with British rule in the years leading up to the American Revolution Janelle alternates between running and walking. She begins by walking for a short period, and then runsfor the same amount of time. She takes a break before beginning to walk again. Consider the four graphsbelow. Which graph best represents the given situation? How would browne have written the following sentence without the hyperbole?I should move heaving and earth-so to speak-until i got [office]. Mrs.smith deposits $980 in a saving account that pays 3.1% interest compounded daily Apr 20, 11:27:10 AMA series of coins are stacked to represent a right circular cylinder (on the left). Thecoins are then "slid" to represent a distorted cylinder (on the right). The samenumber of congruent coins was used in each stack. Which of the following statementswill be TRUE regarding these stacks of coins? HELP HELP HELP MEEEEEEEEE PLEASEEEEEEEEE which is the best estimate for the average rate of change for the quadratic function graph on the interval [tex]0 \leqslant x \leqslant 4[/tex] What happens to Centripedal force on the planet around a star if the radius of an orbit is doubled ? assume that velocity remains the same A) the Centripedal force will be half of its original value B) the Centripedal force will be double its original value C) the Centripedal force will be 1/4 of its original value D) the Centripedal force will not change x = y + 3(2y + x = 12